Study the scatterplot and trend line. Which two points can be used to find the equation of the trend line?

Which points are on the trend line?
(1, 30) and (9, 95)
(2, 30) and (6, 70)
(2, 45) and (8, 90)
(3, 50) and (7, 65)

Answers

Answer 1

Answer:

C

Step-by-step explanation:

Just trust

Answer 2

Answer:

C

Step-by-step explanation:

I did the assignment in edge and got it right.

Proof:


Related Questions

Complete the table for the given rule.
Rule: y is 0.75 greater than x
x y
0
3
9

Answers

The complete table is

x    y

0    0.75

3     3.75

9     9.75

What is equation?

An equation is a mathematical statement that is made up of two expressions connected by an equal sign.

What is substitution?

Substitution means putting numbers in place of letters to calculate the value of an expression or equation.

According to the given question.

We have values of x.

Also, one rule that y is 0.75 greater than x.

So, we have a equation for finding the value of y  i.e.

[tex]y = x + 0.75..(i)[/tex]

For finding the value of y

At x = 0, substitute x = 0 in equation (i)

[tex]y = 0 + 0.75\\\implies y = 0.75[/tex]

At x = 3, substitute x = 3 in equation (i)

[tex]y = 3+0.75\\\implies y = 3.75[/tex]

At x = 9, substitute x = 9 in equation (i)

[tex]y = 9+0.75\\\implies y = 9.75[/tex]

Hence, the complete table is

x    y

0    0.75

3     3.75

9     9.75

Find out more information about equation and substitution here:

https://brainly.com/question/10852714

#SPJ2

Write the additive inverse of each of the following rational numbers:

Answers

Answer:

(1)-3/4, (2) 5/21, (3)4/43

HELPPPPP ASP PLZZZZZ

Answers

Answer:

[tex](f-g)(x)[/tex]

[tex]f(x)-g(x)[/tex]

[tex]x^{2} -6x-27-x+9[/tex]

[tex]x^{2} -7x-18[/tex]

----------------------

[tex](f*g)(x)[/tex]

[tex]=f(x)g(x)[/tex]

[tex](x^{2} -6x-27)(x-9)[/tex]

[tex]=x^{3} -15x^{2}+27x+243[/tex]

----------------------

[tex]\frac{f}{g} (x)[/tex]

[tex]\frac{x^{2} -6x-27}{x-9}[/tex]

[tex]\frac{(x-9)(x+3)}{x-9}[/tex]

[tex]x+3[/tex]

-----------------------

[tex](f+g)(x)[/tex]

[tex]f(x)+g(x)[/tex]

[tex]=x^{2} -6x-27+x-9[/tex]

[tex]=x^{2} -5x-36[/tex]

------------------------

OAmalOHopeO

------------------------

Im new to this app!
And im looking for help!!
Please help ASAP!!!
Please!!!!

Answers

y=x²-10x-7

a>0 so we will be looking for minimum

x=-b/2a=10/2=5

y=25-50-7=-32

Answer: (5;32)

look at the image for the question

Answers

It’s 16
You would multiply height (4) by the base (length(2) x width(2)=4) which is 16

1. A helicopter is at a position from two VORS (VHF Omnidirectional
Radio Range, an aircraft navigation system operating in the VHF band -
not covered in chapter) as in the diagram shown below. Given the angles
shown, find the third angle.
Helicopter
74.0°
66.0°
VOR
VOR

Answers

The position of the helicopter and the two VORs forms a triangle and the third angle formed by these three entities is 40 degrees

The diagram is not shown; however, the question can still be answered.

The given angles are:

[tex]\theta_1 = 74.0^o[/tex]

[tex]\theta_2 = 66.0^o[/tex]

Represent the third angle as [tex]\theta_3[/tex]

The helicopter and the 2 VORs form a triangle.

So, we make use of the following theorem to calculate the third angle

[tex]\theta_1 + \theta_2 + \theta_3= 180^o[/tex] ---- sum of angles in a triangle

Substitute known values

[tex]74.0^o + 66.0^o + \theta_3= 180^o[/tex]

[tex]140.0^o + \theta_3= 180^o[/tex]

Collect like terms

[tex]\theta_3= 180 -140.0^o[/tex]

[tex]\theta_3= 40^o[/tex]

Hence, the third angle is 40 degrees.

Learn more about angles in a triangle at:

https://brainly.com/question/14780489

Find the area of the figure. (Sides meet at right angles.)

Answers

Answer:

56

Step-by-step explanation:

A=(3*4)+(4*(4+3+4))=56

Find an upper bound for E(h) the error of the machine approximation of the two-point forward difference formula for the first derivative and then find the h corresponding to the minimum of E(h).

The two-point forward difference formula for f'(x) is:_________

Answers

Answer:

I doubt it is not going to be a great

Please Help!! Whoever helps and gets it correct gets Brainliest and 5 star rating!!

Answers

Answer:

the reasoning states that "all the numbers begin with a 7 or an 8"

however this is not accurate as they can be in different placements

which can make a big difference in the total estimate.

for example:

the number could've been an 8, or an 80

they both begin with an 8

however have totally different values and could have messed up the total estimated number.

hope this helps :D

Seven and one-half foot-pounds of work is required to compress a spring 2 inches from its natural length. Find the work required to compress the spring an additional 3 inch.

Answers

Answer:

Apply Hooke's Law to the integral application for work: W = int_a^b F dx , we get:

W = int_a^b kx dx

W = k * int_a^b x dx

Apply Power rule for integration: int x^n(dx) = x^(n+1)/(n+1)

W = k * x^(1+1)/(1+1)|_a^b

W = k * x^2/2|_a^b

 

From the given work: seven and one-half foot-pounds (7.5 ft-lbs) , note that the units has "ft" instead of inches.   To be consistent, apply the conversion factor: 12 inches = 1 foot then:

 

2 inches = 1/6 ft

 

1/2 or 0.5 inches =1/24 ft

To solve for k, we consider the initial condition of applying 7.5 ft-lbs to compress a spring  2 inches or 1/6 ft from its natural length. Compressing 1/6 ft of it natural length implies the boundary values: a=0 to b=1/6 ft.

Applying  W = k * x^2/2|_a^b , we get:

7.5= k * x^2/2|_0^(1/6)

Apply definite integral formula: F(x)|_a^b = F(b)-F(a) .

7.5 =k [(1/6)^2/2-(0)^2/2]

7.5 = k * [(1/36)/2 -0]

7.5= k *[1/72]

 

k =7.5*72

k =540

 

To solve for the work needed to compress the spring with additional 1/24 ft, we  plug-in: k =540 , a=1/6 , and b = 5/24 on W = k * x^2/2|_a^b .

Note that compressing "additional one-half inches" from its 2 inches compression is the same as to  compress a spring 2.5 inches or 5/24 ft from its natural length.

W= 540 * x^2/2|_((1/6))^((5/24))

W = 540 [ (5/24)^2/2-(1/6)^2/2 ]

W =540 [25/1152- 1/72 ]

W =540[1/128]

W=135/32 or 4.21875 ft-lbs

Step-by-step explanation:

Round 573.073 to the greatest place

Answers

Answer:

574

Step-by-step explanation:

To round a two-digit number to the nearest ten, simply increase it or decrease it to the nearest number that ends in 0: When a number ends in 1, 2, 3, or 4, bring it down; in other words, keep the tens digit the same and turn the ones digit into a 0

Hope this helps <3

Paul can install a 300-square-foot hardwood floor in 18 hours. Matt can install the same floor in 22 hours. How long would it take Paul and Matt to install the floor working together?
4 hours
9.9 hours
13.2 hours
30 hours

Answers

Answer:

9.9 hours

Step-by-step explanation:

The formula to determine the time together is

1/a+1/b = 1/c  where a and b are the times alone and c is the time together

1/18 + 1/22 = 1/c

The least common multiply of the denominators is 198c

198c(1/18 + 1/22 = 1/c)

11c+ 9c = 198

20c = 198

Divide by 20

20c/20 =198/20

c =9.9

Answer:

B - 9.9 hrs

Step-by-step explanation:

took the test.

find the missing side of the triangle

Answers

Answer:

x = 34

Step-by-step explanation:

Pytago:

x[tex]30^{2} + 16^{2} = x^2\\x = \sqrt{30^2 + 16^2} \\x = 34[/tex]

Find an equation of the plane orthogonal to the line
(x,y,z)=(0,9,6)+t(7,−7,−6)

which passes through the point (9, 6, 0).

Give your answer in the form ax+by+cz=d (with a=7).

Answers

The given line is orthogonal to the plane you want to find, so the tangent vector of this line can be used as the normal vector for the plane.

The tangent vector for the line is

d/dt (⟨0, 9, 6⟩ + ⟨7, -7, -6⟩t ) = ⟨7, -7, -6⟩

Then the plane that passes through the origin with this as its normal vector has equation

x, y, z⟩ • ⟨7, -7, -6⟩ = 0

We want the plane to pass through the point (9, 6, 0), so we just translate every vector pointing to the plane itself by adding ⟨9, 6, 0⟩,

(⟨x, y, z⟩ - ⟨9, 6, 0⟩) • ⟨7, -7, -6⟩ = 0

Simplifying this expression and writing it standard form gives

x - 9, y - 6, z⟩ • ⟨7, -7, -6⟩ = 0

7 (x - 9) - 7 (y - 6) - 6z = 0

7x - 63 - 7y + 42 - 6z = 0

7x - 7y - 6z = 21

so that

a = 7, b = -7, c = -6, and d = 21

An equation of the plane orthogonal to the line 7x - 7y - 6z = 21.

The given line is orthogonal to the plane you want to find,

So the tangent vector of this line can be used as

The normal vector for the plane.

The tangent vector for the line is,

What is the tangent vector?

A tangent vector is a vector that is tangent to a curve or surface at a given point.

d/dt (⟨0, 9, 6⟩ + ⟨7, -7, -6⟩t ) = ⟨7, -7, -6⟩

Then the plane that passes through the origin with this as its normal vector has the equation

⟨x, y, z⟩ • ⟨7, -7, -6⟩ = 0

We want the plane to pass through the point (9, 6, 0), so we just

translate every vector pointing to the plane itself by adding ⟨9, 6, 0⟩,

(⟨x, y, z⟩ - ⟨9, 6, 0⟩) • ⟨7, -7, -6⟩ = 0

Simplifying this expression and writing it in standard form gives

⟨x - 9, y - 6, z⟩ • ⟨7, -7, -6⟩ = 0

7 (x - 9) - 7 (y - 6) - 6z = 0

7x - 63 - 7y + 42 - 6z = 0

7x - 7y - 6z = 21

So that, a = 7, b = -7, c = -6, and d = 21.

To learn more about the equation of plane visit:

https://brainly.com/question/1603217

State if the scenario involves a permutation or a combination. Then find the number of possibilities.

A team of 15 basketball players needs to choose two players to refill the water cooler.

Permutation/Combination:

Answer:

Answers

Answer:

Permutation ; 210 ways

Step-by-step explanation:

Permutation and combination methods refers to mathematical solution to finding the number of ways of making selection for a group of objects.

Usually, selection process whereby the order of selection does not matter are being treated using permutation, while those which takes the order of selection into cognizance are calculated using combination.

Here, selecting 2 players from 15 ; since order does not matter, we use permutation ;

Recall :

nPr = n! ÷ (n - r)!

Hence,

15P2 = 15! ÷ (15 - 2)!

15P2 = 15! ÷ 13!

15P2 = (15 * 14) = 210 ways

Reason Can you subtract a positive integer from a positive integer
and get a negive result? Explain your answer.

Answers

Answer:

No

Step-by-step explanation:

No matter the situation, when you multiply a negative by a negativeyou get a positive and a positive by a positive you get a positive. but if its two different like a negative and a positive then its NEGITIVE.

let's say you have 23 and you're multiplying by 2.

It's always increasing so it doesnt ever reach the negitive numbers.

Find the length of the arc.

A. 539π/12 km
B. 9π/3 km
C. 9π/2 km
D. 18π km

Answers

Answer:

b because it is I found out cus I took test

The length of the arc 9π/2 km.

The answer is option C.9π/2 km.

What is the arc of the circle?

The arc period of a circle can be calculated with the radius and relevant perspective using the arc period method.

  ⇒angle= arc/radius

     ⇒  135°=arc/6km

     ⇒ arc =135°*6km

     ⇒arc=135°*π/180° * 6km

    ⇒arc = 9π/2 km

Learn more about circle here:-https://brainly.com/question/24375372

#SPJ2

Find the slope of the line that goes through the
(2,6) and (-1, -6)

Answers

We can use the formula y2-y1/x2-x1 to get our slope. y2 and x2 are our second y and x coordinates, meanwhile y1 and x1 are our first y and x coordinates. -6-6/-1 -2 is -12/-3. -12/-3 is 4, the slope is 4.
the slope is 1/4 because you use the slope intercept formula

please solve the question ​

Answers

Answer:

[tex]g(-1) = -1[/tex]

[tex]g(0.75) = 0[/tex]

[tex]g(1)= 1[/tex]

Step-by-step explanation:

Given

See attachment

Solving (a): g(-1)

We make use of:

[tex]g(x) = -1[/tex]

Because: [tex]-1 \le x < 0[/tex] is true for x =-1

Hence:

[tex]g(-1) = -1[/tex]

Solving (b): g(0.75)

We make use of:

[tex]g(x) = 0[/tex]

Because: [tex]0 \le x < 1[/tex] is true for x =0.75

Hence:

[tex]g(0.75) = 0[/tex]

Solving (b): g(1)

We make use of:

[tex]g(x) = 1[/tex]

Because: [tex]1 \le x < 2[/tex] is true for x =1

Hence:

[tex]g(1)= 1[/tex]

Solve for x: 10/3 = x/(−5/2)

Answers

9514 1404 393

Answer:

  x = -25/3

Step-by-step explanation:

Multiply by the inverse of the coefficient of x. Reduce the fraction.

  (-5/2)(10/3) = (-5/2)(x/(-5/2))

  -50/6 = x = -25/3

Answer:

-25/3

Step-by-step explanation:

the other person is also correct. khan said so

Which ratio is equal to 27 : 81?

Answers

3:9 and if you reduce it again, 1:3

Answer:

1:3

Step-by-step explanation:

27 : 81

Divide each side by 27

27/27 : 81/27

1:3

Identify the transformed function that represents f(x) = ln x stretched vertically by a factor of 17, reflected across the x-axis, and shifted by 19 units left.
A. g(x) = −17ln (x + 19)
B. g(x) = 17ln (x − 19)
C. g(x) = 17ln (x + 19)
D. g(x) = −17ln (x − 19)

Answers

Answer:

b

Step-by-step explanation:

ANSWER. EXPLANATION. The given logarithmic function is. The transformation,. stretches the graph of y=f(x) vertically by a factor of c units ...

4 votes

ANSWER[tex]y = - 3 ln(x - 7) [/tex]EXPLANATIONThe given logarithmic function is [tex]f(x) = ln(x) [/tex]The transformation, [tex]y = - cf(x - k)[/tex]stretches

The length of a rectangle is twice its width. If the area of the rectangle is 72in², find its perimeter

Answers

Let breadth be x

Length=2x

[tex]\\ \sf\longmapsto Area=Length\times Breadth[/tex]

[tex]\\ \sf\longmapsto 72=2x(x)[/tex]

[tex]\\ \sf\longmapsto 2x^2=72[/tex]

[tex]\\ \sf\longmapsto x^2=\dfrac{72}{2}[/tex]

[tex]\\ \sf\longmapsto x^2=36[/tex]

[tex]\\ \sf\longmapsto x=\sqrt{36}[/tex]

[tex]\\ \sf\longmapsto x=6[/tex]

Length=6×2=12inBreadth=6in

[tex]\\ \sf\longmapsto Perimeter=2(L+B)[/tex]

[tex]\\ \sf\longmapsto Perimeter=2(12+6)[/tex]

[tex]\\ \sf\longmapsto Perimeter=2(18)[/tex]

[tex]\\ \sf\longmapsto Perimeter=36in[/tex]

Graph y=|x|+5, how does it compare to parent graph y=|x|

Answers

9514 1404 393

Answer:

  it is shifted 5 units upward

Step-by-step explanation:

The y-coordinate is a measure of the distance above the x-axis. When 5 is added to a y-coordinate, the point is shifted 5 units upward.

The function y = |x| +5 adds 5 units to the y-value of every point of the graph of y = |x|. The graph of y=|x|+5 is shifted 5 units upward from the parent graph.

In a class of 70 pupils, 36 like tasty time , 34 like ice-
cream, 6 like both tasty time }
draw a Venn diagram to show the data.
find how
many
like neither tasty time nor ice-cream

Answers

Step-by-step explanation:

I think this might be the correct answer

The number of pupils that like neither tasty-time nor ice cream is 6 if in a class of 70 pupils, 36 like tasty time, 34 like ice cream, 6 like both tasty times.

What is the Venn diagram?

It is defined as the diagram that shows a logical relation between sets.

The Venn diagram consists of circles to show the logical relation.

We have:

In a class of 70 pupils, 36 like tasty time, 34 like ice cream, 6 like both tasty time.

Total = 70 pupils

Number of like tasty time = 36

Number of like ice cream = 34

Number of like both = 6

Let x be the total number of pupils that like neither tasty-time nor ice cream

The number of pupils that like ice cream only =  34 - 6 = 28

The number of pupils that like tasty-time only = 36 - 6 = 30

From the Venn diagram:

28 + 30 + 6 + x = 70

x = 70 - 64

x = 6

Thus, the number of pupils that like neither tasty-time nor ice cream is 6 if in a class of 70 pupils, 36 like tasty time, 34 like ice cream, 6 like both tasty times.

Learn more about the Venn diagram here:

brainly.com/question/1024798

#SPJ2

Find the length represented by x for each pair of similar triangles.
18cm, 9cm, and x
30cm, 15cm, and 25cm

Answers

Answer:

15 cm

Step-by-step explanation:

Since the traingles are similar, we can find the ratio between the side lengths, and it will be the same for each side.

We can use the side length 9 and 15 to find this ratio. 15/9=5/3. So, the ratio of a side length of the larger triangle to the smaller one is 5/3, so our equation becomes 5/3 = 25/x. Use any method you like to find that x=15.

Hope this helped,

~cloud

A bookkeeper needs to post the cost of the desk and the chair into his records. The cost of the desk is fives times the cost if the chair. The total cost of the desk and the chair is $720, what is the cost of the chair?

Answers

Answer:

120

Step-by-step explanation:

720÷6

why?

becoz 6= 1+5

1 is the cosy of the chair

5 is the cost of the desk

An article is marked to sell at a gain of 10%. If it be sold for Rs. 7.50 less there would be loss of 5%, find the cost price. ​

Answers

Answer:

750,. is answer

I hope it helps

"What mathematical ideas are you curious to know more about as a result of takingthis class? Give one example of a question about the material that you'd like to explorefurther, and describe why this is an interesting question to you."

Answers

One mathematical idea I've always been curious about is "integration and derivation".

Integration is about assimilating different variables. Derivation is a mathematical process whereby a result is gotten from some initial assumptions.

Integration is used everyday in different aspects of our lives. For example, if a person is travelling from let's say point A to point B, the speed used by the person might vary but through integration, one can easily get the accurate speed.

Through the division of equations into smaller bits, once can use integration to get the answer that one seeks. Architect can use integration in building the right structures at the exact places where the structures fits.

Likewise derivatives can be used by businesses in assessing whether a profit or loss will be made for a particular transaction or sale of product.

You can read more on:

https://brainly.com/question/14295614

You are dealt one card from a​ 52-card deck. Find the probability that you are not dealt a heart.

The probability is ___.
​(Type an integer or a fraction. Simplify your​ answer.)

Answers

Answer:

3/4

Step-by-step explanation:

There are 13 hearts in a 52 deck.

52-13=39

39/52=3/4

The probability that you are not dealt a heart from the deck of cards is 3/4.

What is the probability that you are not dealth with a heart?

Probability determines the chances that an event would occur. The probability the event occurs is 1 and the probability that the event does not occur is 0.

The probability that you are not dealth with a heart = 1 - (number of hearts / total number of cards)

1 - 13/52 = 39/52 =  3/4

To learn more about probability, please check: https://brainly.com/question/13234031

Other Questions
Describe your class teacher what qualities did settlers need to survive on the great plains can you suggest me some biology experimental project work topics? Mrs. Moreau was planning to have several guests at her home for a traditional Thanksgiving dinner. She had cooked Shady Brook Farms fresh turkeys in the past and had enjoyed them very much. When she went to her usual grocery store, she discovered that the store no longer carried the Shady Brook Farms brand. She called several other grocery stores and was finally able to locate Shady Brook Farms fresh turkeys at a small grocery store approximately 10 miles away. She drove to the store and bought a 20-pound Shady Brook Farms turkey, even though the price per pound was higher than what she normally paid at her usual grocery store. For Mrs. Moreau, the Shady Brook Farms turkey was a(n) Factor this polynomial expression.3x^2 - 12x+ 12A. (3x - 2)(x-6)B. 3(x-2)(x + 2)C. 3(x-2)(x-2)D. 3(x + 2)(x + 2) Does this graph represent a function (scatter) A. Yes B. No HELP PLEASEIn which of the following scenarios would using a GIS be helpful? assessing the soil erosion risk within a watershedproviding navigation assistance to guide a lost driver to his destinationtranscribing inaccurate historical data into accurate historical mapscalculating local time when crossing the International Date Lineanalyzing the relationship between functional and formal regions Ok, so I'm trying to read piano sheet music, but I'm a bit confused. In the pic, can you guys tell me what they are and how does one play them on the keyboard? Like do you play one bar with the left hand and the other with the right hand? If so which bar goes with which hand? Dialogue between Michael and an educator about his lifestyle and his Choice of studying at an FET College and eating habits under which Language groups does Nepali language fall? Write briefly about it's. What is the slope of a line parallel to line B? Which of the following is a vector quantity? i. Force ii. Velocity iii. Acceleration iv. All of these 5771 what movie is this?? 9. During an experiment the students prepared three mixtures A)Starch in water B) Sodium chloride solution C) Tincture of Iodine. i) Students observed a visible beam of light through mixture A. Why? ii) Tincture of lodTe did not show Tyndall effect. Explain reason. ill) How can you relate particle size to Tyndall effect? Please hurry I will mark you brainliest What is the value of p so that the line passing through (6, 2) and (9, p) has a slope of -1? Using the diagram below, which of the following parts of the triangles arecongruent?7 milesB16 ft12 ftD21 ftE1 mile = 5280 ftGiven:1. ACAB-ACED2. AB ||EDA. ZA ZBB. ZASZCy could someone please answer this Find the length of AD in the figure.A. 26 unitsB. 34 unitsC. 122 unitsD. 130 units Sue was paid $384 for working 32 hours. How many hours will she have to work to earn $672? What is the volume of the solid? Let =3.14